Wenn Sie einen allgemeinen Lorentz-Boost unter Verwendung eines xxx-Achsen-Boosts konstruieren, wie ist die zweite Rotation im Verhältnis zur ersten Rotation?

Wie in dieser Frage und dieser anderen Frage diskutiert , ist es möglich, Lorentz-Boosts entlang einer beliebigen Richtung zu konstruieren, indem nur der Lorentz-Boost entlang verwendet wird X -Achse, indem Sie das folgende Verfahren durchführen:

(1) Drehen Sie die Koordinatenachsen, um die auszurichten X -Achse mit Richtung des Boosts.

(2) Führen Sie einen Boost entlang der neuen durch X -Achse mit der üblichen Formel.

(3) Zurückdrehen.

Das ist alles schön und gut, bis auf den letzten Schritt. Was bedeutet "Rotate back." eigentlich gemeint? Welche Beziehung besteht zwischen dem Rest des Prozesses und dem letzten Schritt?

In dieser oben verlinkten ersten Frage behauptet die akzeptierte Antwort ohne Begründung, dass (zumindest in 2 räumlichen Dimensionen) die zweite Drehung lediglich die Umkehrung der ersten Drehung ist. Das erscheint mir intuitiv plausibel.

Die zweite oben verlinkte Frage und ihre akzeptierte Antwort legen nahe, dass die beiden Rotationen im Allgemeinen - in 3 räumlichen Dimensionen - nicht invers zueinander sind. Die verknüpfte Antwort erklärt jedoch nicht, welche Beziehung zwischen ihnen besteht.

Nehmen B ( v N ^ ) ein Schub entlang der sein N ^ Richtung der Größe v Und R ( θ k ^ ) eine Drehung des Winkels sein θ um die durch gegebene Achse k ^ Wir können das Problem wie folgt symbolisieren:

B ( v N ^ ) = R ( θ ( v , N ^ ) k ^ ( v , N ^ ) ) B ( v X ^ ) R ( cos 1 ( X ^ N ^ ) X ^ × N ^ | X ^ × N ^ | )

Die Frage ist also: Was ist die funktionale Form von θ ( v , N ^ ) k ^ ( v , N ^ ) was gibt den Drehwinkel und die Drehachse für die zweite Drehung an?

Du hast Recht; es ist nicht "die umgekehrte Drehung" im einfachen Sinne. Das Gesamtergebnis ist ziemlich kompliziert, nicht zuletzt, weil man, um eine Drehung als Matrix aufzuschreiben, zuerst einige Achsen angeben muss, und es gibt mehr als eine vernünftige Wahl, wenn auch Lorentz-Boosts im Spiel sind.
@AndrewSteane Können Sie eine Referenz angeben, die die Lösung bespricht?
Verzeihung; Ich kenne so einen Referenten nicht. Mein Kommentar basiert auf meinen Bemühungen, die Lösung selbst zu schreiben. Ich bin so weit gekommen, festzustellen, was ich hier behaupte. Natürlich kann man immer schreiben Λ = R 1 Λ z R 2 und finde die Matrizen; was schwierig ist, ist, sie zu interpretieren.
@AndrewSteane Ich habe mehrere Stunden damit verbracht, über dieses Problem nachzudenken und die Algebra auszuarbeiten. Es stellt sich heraus, dass die Rotationsmatrizen tatsächlich invers sind. Ich habe hier in diesem Thread eine ausführliche Antwort gepostet, falls Sie daran interessiert sind.
OK; Es scheint, dass mein Kommentar zur umgekehrten Drehung etwas irreführend war. Das tut mir leid. Ich kann mich jetzt nur schwer erinnern, was das Problem war, worüber ich damals nachgedacht habe. Es ist ungefähr so, dass die zweite Drehung relativ zur Schubrichtung für einen Beobachter, der im Rahmen sitzt und sich in diese Richtung bewegt, leicht zu interpretieren ist, aber man muss vorsichtig sein, welchen Winkel dies relativ zum ursprünglichen Satz von Achsen ist .
@AndrewSteane Hmm, sicher, wenn Sie ein Problem mit meiner Antwort bemerken, lassen Sie es mich bitte wissen. Ich fühle mich ein wenig dumm, weil ich den Hauptpunkt der Frage nicht früher erkannt habe (nämlich, dass die Antwort ziemlich offensichtlich ist, da wir uns in einem linearen Vektorraum befinden). Außerdem enthält die Math StackExchange-Frage, auf die ich verlinke, viele interessante Randnotizen zu Rotationen, die wirklich geholfen haben.

Antworten (2)

Geben Sie hier die Bildbeschreibung ein

Sie können die räumliche Lorentz-Transformation erhalten, indem Sie zwei Rotationen anwenden.

Wir wollen die x-Achsen auf die x'-Achsen ausrichten, dies kann durch zwei Drehungen erfolgen, zuerst um die z-Achsen mit dem Winkel drehen φ und dann mit dem Engel um die neue y-Achse rotieren ψ . daher:

die Transformationsmatrix um die z-Achsen lautet:

S z = [ 1 0 0 0 0 cos ( φ ) Sünde ( φ ) 0 0 Sünde ( φ ) cos ( φ ) 0 0 0 0 1 ]

und über die neuen y-Achsen ist:

S j = [ 1 0 0 0 0 cos ( ψ ) 0 Sünde ( ψ ) 0 0 1 0 0 Sünde ( ψ ) 0 cos ( ψ ) ]

mit :

φ = arctan ( v j v X )
ψ = arctan ( v X v X 2 + v j 2 )
und der Boost-Vektor
v = [ v X v j v z ]
erhält man die räumliche Lorentz-Transformation:

L D = S z S j L S j T S z T
mit der Lorentztransformation L

L = [ γ γ v 0 0 γ v γ 0 0 0 0 1 0 0 0 0 1 ]

L D = [ γ γ v X γ v j γ v z v X γ 2 γ v X 2 + v j 2 + v z 2 v 2 ( γ 1 ) v j v X v 2 v X v z ( γ 1 ) v 2 v j γ 2 ( γ 1 ) v j v X v 2 v X 2 + v j 2 γ + v z 2 v 2 v j v z ( γ 1 ) v 2 v z γ 2 v X v z ( γ 1 ) v 2 v j v z ( γ 1 ) v 2 v z 2 γ + v X 2 + v j 2 v 2 ] = [ γ γ v γ v ICH 3 + γ 1 v 2 v v T ]

und die inverse Lorentztransformation lautet:

L D 1 = L D ( v v ) = [ γ γ v γ v ICH 3 + γ 1 v 2 v v T ]

Wo ICH 3 ist ein 3 × 3 Einheitsmatrix.

bearbeiten

Was bedeutet "Rotate back." eigentlich gemeint?

Beispiel:

Die Komponenten des Drehimpulsvektors im Inertialsystem sind:

(1) ( L ) ICH = [ B ICH S ] ( ICH ) B ( ω ) B

wobei B der Body-Frame-Index und I der Trägheits-Frame-Index ist. [ B ICH S ] ist die Transformationsmatrix zwischen Body-Frame und Inertial-Frame. ( ICH ) B ist der 3 × 3 Trägheitstensor im Body-Frame.

Wenn nun die Winkelvektorkomponenten im Inertial-Frame so angegeben sind:

( ω ) B = [ ICH B S ] ( ω ) ICH

und Gleichung (1) :

( L ) ICH = [ B ICH S ] ( ICH ) B [ ICH B S ] ( ω ) ICH = S ( ICH ) B S T ( ω ) ICH

Die "Komponenten" des Trägheitstensors werden umgeformt

( ICH ) ICH = S ( ICH ) B S T

Dasselbe gilt für jede Matrixtransformation wie die Lorentz-Matrix.

Das beantwortet die Frage nicht wirklich. Sie haben angenommen, dass die richtige Form der zweiten Drehung die Umkehrung der ersten ist, aber Sie haben das nicht begründet oder bewiesen. Auch das analogistische Argument bezüglich des Trägheitstensors ist nicht überzeugend. Lorentz-Boosts und Trägheitstenöre haben einen anderen Rang, arbeiten in metrischen Räumen mit unterschiedlichen Signaturen, und es ist nicht einmal ganz offensichtlich, dass sie denselben Tensorcharakter haben.
Sie haben angenommen, dass die korrekte Form der zweiten Drehung die Umkehrung der ersten ist? Das ist nicht korrekt, die erste Rotation hat nichts mit der zweiten Rotation zu tun!!!
Es muss eindeutig eine Beziehung haben. Die Beziehung, die Sie in Ihrer Antwort postulieren, ist die R 2 = R 1 T seit S z S j = ( S j T S z T ) T .
S j S j T = ICH 3 Und S z S z T = ICH 3 somit ist die Lorentz-Matrix bedingt L D Gleichung, also warum denkst du, ist etwas falsch? L D = R L R T Wo R = S z S j Und R T = S j T S z T aber diese reine Mathematik? was ist falsch an dieser gleichung?
Das Problem ist, dass Sie nicht wirklich bewiesen haben, dass Ihre Form von L D ist richtig. Können Sie eine Referenz für diese Behauptung angeben? Das ist alles, wonach ich suche.
Tut mir leid, ich kann nicht, ich habe dieses Problem vor langer Zeit selbst gelöst

Die Lösung ist viel einfacher als es scheint. Das Ergebnis des ersten Links, den Sie angeben, ist sogar im Allgemeinen wahr, und der Vorschlag im zweiten Link, dass die Rotationen nicht zusammenhängen, ist falsch. Im Allgemeinen ist die Beziehung zwischen den beiden Rotationen in Λ = R 2 Λ X R 1 ist das R 1 Und R 2 sind Inverse (d. h. Transponierte) voneinander.

Nun, lassen Sie uns diese Antwort begründen.

Alle Operationen, denen wir in diesem Problem begegnen, sind endlichdimensional (dh 4-dimensional) und linear, was bedeutet, dass sie dargestellt werden können als 4 × 4 Matrizen. Im Allgemeinen gibt es zwei Interpretationen, die einer nicht-singulären quadratischen Matrix gegeben werden können: (1) es ist ein Basiswechsel von einem Koordinatensystem zu einem anderen, oder (2) es ist eine lineare Transformation, die Vektoren in einen linearen Vektor abbildet Raum zu anderen Vektoren im selben Raum. In diesem Problem interpretieren wir offensichtlich die Rotationen als Basisänderungen und den Boost als lineare Transformation .

Gegeben zwei Basen A Und B für einen linearen Vektorraum und eine lineare Transformation T auf diesem Platz ist das bekannt T 's Darstellungen in den beiden verschiedenen Basen sind verwandt durch T B = U A B T A U B A (Wo U A B ist die Basiswechselmatrix aus A Zu B ). Das ist per Definition klar U A B = ( U B A ) 1 . Für jede Rotationsmatrix R , R 1 = R T ; Daher ist es nun offensichtlich, dass ein Lorentz-Boost entlang einer beliebigen Achse gegeben werden kann R T Λ X R

Dies beantwortet die gestellte Frage, aber lassen Sie uns der Konkretheit halber das allgemeine Ergebnis für einen beliebigen Lorentz-Boost mit dieser Methode herleiten.

Zuerst verwenden wir die in dieser Math StackExchange-Antwort bereitgestellte Formel , um die Form der Rotationsmatrizen zu berechnen.

Da wir den Einheitsvektor drehen wollen N ^ =< N X , N j , N z > in den Einheitsvektor X ^ =< 1 , 0 , 0 > , wir bekommen N ^ X ^ = cos ( θ ) = N X Und N ^ × X ^ =< 0 , N z , N j > . Das gibt

[ v ] × = [ 0 N j N z N j 0 0 N z 0 0 ]

Daher erhalten wir aus der Math StackExchange-Antwort

R = ICH + [ v ] × + [ v ] × 2 1 1 + cos ( θ )

und wir leiten das ab 4 × 4 Rotationsmatrix ist

R ( N ^ , X ^ ) = [ 1 0 0 0 0 N X N j N z 0 N j 1 N j 2 1 + N X N j N z 1 + N X 0 N z N j N z 1 + N X 1 N z 2 1 + N X ]

Als Anmerkung, R T ( N ^ , X ^ ) = R ( X ^ , N ^ ) wie erwartet.

Die Matrix für die X -Achse Lorentz-Boost ist

Λ ( β X ^ ) = [ γ γ β 0 0 γ β γ 0 0 0 0 1 0 0 0 0 1 ]

Dies führt zur endgültigen Berechnung des Matrixprodukts

Λ ( β N ^ ) = R T ( N ^ , X ^ ) Λ ( β X ^ ) R ( N ^ , X ^ )

Nach einiger langwieriger Algebra ist das Endergebnis

Λ ( β N ^ ) = [ γ γ β N X γ β N j γ β N z γ β N X 1 + ( γ 1 ) N X 2 ( γ 1 ) N X N j ( γ 1 ) N X N z γ β N j ( γ 1 ) N X N j 1 + ( γ 1 ) N j 2 ( γ 1 ) N j N z γ β N z ( γ 1 ) N X N z ( γ 1 ) N j N z 1 + ( γ 1 ) N z 2 ]

das ist (Modulo-Notation) diese Boost-Matrix , die das Standardergebnis ist, das z. B. in Jackson zitiert wird .